Automorphisms are isomorphisms

  • Thread starter Thread starter Dustinsfl
  • Start date Start date
Click For Summary
SUMMARY

The discussion focuses on proving the relationship between automorphisms and conjugation in group theory, specifically showing that if \(\sigma \in Aut(G)\) and \(\varphi_g\) represents conjugation by \(g\), then \(\sigma \varphi_g \sigma^{-1} = \varphi_{\sigma(g)}\). This leads to the conclusion that the inner automorphisms \(Inn(G)\) form a normal subgroup of the automorphism group \(Aut(G)\). Key terms such as "conjugation," "automorphisms," and "normal subgroup" are defined and clarified throughout the conversation.

PREREQUISITES
  • Understanding of group theory concepts, specifically automorphisms and isomorphisms.
  • Familiarity with the notation and definitions of inner automorphisms (Inn) and normal subgroups (trianglelefteq).
  • Knowledge of the properties of homomorphisms and their implications in group operations.
  • Basic understanding of conjugation in the context of group actions.
NEXT STEPS
  • Study the properties of automorphisms in group theory, focusing on the structure of \(Aut(G)\).
  • Learn about inner automorphisms and their significance in understanding group symmetries.
  • Explore the concept of normal subgroups and their role in group theory.
  • Investigate the implications of homomorphisms in group operations, particularly in relation to conjugation.
USEFUL FOR

This discussion is beneficial for students and researchers in abstract algebra, particularly those studying group theory, automorphisms, and their applications in mathematical structures.

Dustinsfl
Messages
2,217
Reaction score
5
If \sigma\in Aut(G) and \varphi_g is conjugation by g prove \sigma\varphi_g\sigma^{-1}=\varphi_{\sigma(g)}. Deduce Inn(G)\trianglelefteq Aut(G)

Let x\in G.

\sigma\varphi_g\sigma^{-1}(x)=\sigma(g\sigma^{-1}(x)g^{-1})=\sigma(g)x\sigma(g)^{-1}

Why is this:
\sigma(g\sigma^{-1}(x)g)=\sigma(g)x\sigma(g)^{-1}
 
Last edited:
Physics news on Phys.org


Dustinsfl said:
If \sigma\in Aut(G) and \varphi_g is conjugation by g prove \sigma\varphi_g\sigma^{-1}=\varphi_{\sigma(g)}. Deduce Inn(G)\trianglelefteq Aut(G)
You should include more information when you ask questions like this. Perhaps I suck more than most at abstract algebra terminology, but I have no idea what Inn and trianglelefteq means, and I'm not sure "conjugation" means what I'm guessing it means either.

Dustinsfl said:
Let x\in G.

\sigma\varphi_g\sigma^{-1}(x)=\sigma(g\sigma^{-1}(x)g)=\sigma(g)x\sigma(g)^{-1}
OK, it looks like my guess about "conjugation" was right. You meant \varphi_g(h)=ghg^{-1}, right? (You missed a ^{-1}).

Dustinsfl said:
Why is this:
\sigma(g\sigma^{-1}(x)g)=\sigma(g)x\sigma(g)^{-1}
Because Aut(G) is the group of automorphisms on G. Automorphisms are permutations that preserve the group multiplication operation.
 


Fredrik said:
You should include more information when you ask questions like this. Perhaps I suck more than most at abstract algebra terminology, but I have no idea what Inn and trianglelefteq means, and I'm not sure "conjugation" means what I'm guessing it means either. OK, it looks like my guess about "conjugation" was right. You meant \varphi_g(h)=ghg^{-1}, right? (You missed a ^{-1}).Because Aut(G) is the group of automorphisms on G. Automorphisms are permutations that preserve the group multiplication operation.

Inn is inner automorphism. Correct on conjugation.
trianglelefteq is a normal subgroup.

I still don't see how that is equal.
 


I assume that you know that automorphisms are isomorphisms, that isomorphisms are homomorphisms, and that every homomorphism f satisfies f(xy)=f(x)f(y) for all x,y in the group? So what is f(xyz)?

What you actually need to evaluate is of the form f(xyz-1), but it's easy to prove that f(x-1)=f(x)-1 for all x in the group.
 
Question: A clock's minute hand has length 4 and its hour hand has length 3. What is the distance between the tips at the moment when it is increasing most rapidly?(Putnam Exam Question) Answer: Making assumption that both the hands moves at constant angular velocities, the answer is ## \sqrt{7} .## But don't you think this assumption is somewhat doubtful and wrong?

Similar threads

Replies
12
Views
2K
  • · Replies 2 ·
Replies
2
Views
2K
  • · Replies 11 ·
Replies
11
Views
3K
  • · Replies 17 ·
Replies
17
Views
3K
  • · Replies 1 ·
Replies
1
Views
2K
  • · Replies 2 ·
Replies
2
Views
1K
  • · Replies 13 ·
Replies
13
Views
1K
  • · Replies 8 ·
Replies
8
Views
2K
  • · Replies 11 ·
Replies
11
Views
2K
  • · Replies 2 ·
Replies
2
Views
2K